Jump to content

dark templar's Content

There have been 1000 items by dark templar (Search limited from 06-06-2020)



Sort by                Order  

#500805 $\sum_{n=1}^{\infty }\frac{(-1)^...

Posted by dark templar on 22-05-2014 - 20:34 in Giải tích

Em nhớ là điều kiện cần để chuỗi $\sum_{n=1}^{\infty} u_n$ hội tụ là $\lim u_n=0$, ngược lại, điều kiện đủ để chuỗi phân kỳ là giới hạn đó khác $0$ :D

Anh nghĩ cái này phải ngược lại chứ nhỉ ? Nếu viết chính xác phải là Chuỗi hội tụ thì $\lim |u_n|=0$ chứ làm gì có $\lim |u_n|=0$ thì chuỗi hội tụ ?




#500696 $\sum_{n=1}^{\infty }\frac{(-1)^...

Posted by dark templar on 22-05-2014 - 09:42 in Giải tích

Em nghĩ chuỗi này không hội tụ.

 

Chứng minh.

$$n^2\ln\left ( 1+\frac{1}{n} \right )=n-\frac{1}{2}+O\left ( \frac{1}{n} \right )$$

 

$$\frac{\left ( 1+n \right )^{n^2}}{e^nn^{n^2}}=\frac{1}{e^n}\left ( 1+\frac{1}{n} \right )^{n^2}=e^{-\frac{1}{2}+O\left ( \frac{1}{n} \right )}\neq 0$$

Cái này là theo định lý nào nhỉ,nếu được em có thể viết rõ giùm anh....

 

Thực ra bài toán gốc là tìm miền hội tụ của chuỗi $\sum_{n=1}^{\infty }\frac{(n+1)^{n^{2}}x^{n}}{n^{n^{2}}}$




#500200 $\sum_{n=1}^{\infty }\frac{(-1)^...

Posted by dark templar on 19-05-2014 - 23:19 in Giải tích

Bài toán: Chuỗi $\sum_{n=1}^{\infty }\frac{(-1)^{n}(n+1)^{n^{2}}}{e^{n}n^{n^{2}}}$ hội tụ hay phân kỳ ?




#498793 Tính $\lim_{n\rightarrow +\infty }\sum_...

Posted by dark templar on 13-05-2014 - 17:53 in Dãy số - Giới hạn

Cho dãy số $(x_n)$ thỏa $x_0=0,x_1=1,x_{n+2}=x_{n+1}+x_n$ với mọi số tự nhiên $n$.

Tính giới hạn :

$$\underset{n\rightarrow +\infty }{\lim}\left ( x_0+\dfrac{x_1}{2012} +\dfrac{x_2}{2012^2}+...+\dfrac{x_n}{2012^n}\right )$$

Cái này nhớ trên diễn đàn có làm 1 lần rồi mà không kiếm được link,chán quá :( Mà lại lười gõ Latex đầy đủ nên đưa hướng dẫn tạm vậy.

 

Xét chuỗi $f(t)=\sum_{k=0}^{\infty }x_{k}t^{k}$ với $x_{k}$ là số hạng thứ $k$ trong dãy số trên.

 

Để ý rằng:

$f(t)=x_{0}+x_1t+\sum_{k=0}^{\infty }x_{k+2}t^{k+2} $

$=t+\sum_{k=0}^{\infty }(x_{k}+x_{k+1})t^{k+2}$

$=t+t^{2}f(t)+tf(t)-x_0t^{0} =t+t^{2}f(t)+tf(t)$

$\Rightarrow f(t)=\frac{t}{1-t^{2}-t}$

 

Và tổng giới hạn trên sẽ có kết quả là $f\left(\frac{1}{2012} \right)$.




#492973 $\sum_{k=0}^{\left \lfloor \frac...

Posted by dark templar on 14-04-2014 - 21:43 in Tổ hợp và rời rạc

Tính tổng

$$\sum_{k=0}^{\left \lfloor \frac{n}{2} \right \rfloor - 2}\binom{n}{k}$$

Thấy có bài này vui vui,ngồi làm thử vậy :P

 

Xét trường hợp $n$ chẵn,$n$ lẻ giải tương tự.Giả sử $n=2m$,ta có:

$S=\sum_{k=0}^{\left\lfloor \frac{n}{2} \right\rfloor -2}\binom{n}{k}=\sum_{k=0}^{m-2}\binom{2m}{k}=\sum_{k=2}^{m}\binom{2m}{m+k}$

 

Giờ thì:

$\sum_{k=0}^{2m}\binom{2m}{k}=4^{m}$

 

Nên:

$2S=4^{m}-\binom{2m}{m}-\binom{2m}{m+1}-\binom{2m}{m-1}$

$\iff S=2^{2m-1}-\binom{2m}{m+1}-\frac{1}{2}\binom{2m}{m}$.

 

P.s: Em chưa giải thử trường hợp $n$ lẻ nên không đưa ra kết quả tổng quát được anh Thế nhé !

Lâu quá không gõ Latex nên em cũng quên mất canh dòng sao rồi....




#492964 Một vài kỹ thuật tính toán với tổng $\sum\limits_{k=m}^n f(k)...

Posted by dark templar on 14-04-2014 - 21:30 in Các dạng toán THPT khác

Cho  $n$ là một số nguyên dương. Chứng minh rằng:

\[ \binom{n}{0}^{-1} + \binom{n}{1}^{-1} + \cdots + \binom{n}{n}^{-1} = \frac{n+1}{2^{n+1}} \left( \frac{2}{1} + \frac{2^2}{2} + \cdots + \frac{2^{n+1}}{n+1} \right).  \]

Bài này tham khảo ở đây (#3).Trong đường link có cả bài tổng quát nữa,nhưng bài giải bằng tiếng Anh nên chú chịu khó đọc nhé !




#492949 Chào mừng 60 năm Chiến thắng Điện Biên Phủ

Posted by dark templar on 14-04-2014 - 21:04 in Thông báo tổng quan

Dạo này cũng chẳng rảnh rang gì cả .Thôi thì không có "sức" thì em ủng hộ bằng tinh thần vậy  :namtay Mong VMF sẽ có thêm 1 chuyên đề mới ra mắt tháng 4 này.




#430639 CM tồn tại số nguyên dương $m$ sao cho $a_m=4$ và $a...

Posted by dark templar on 25-06-2013 - 23:18 in Dãy số - Giới hạn

Bài toán: Cho dãy số nguyên dương $\{a_n \}_{1}^{\infty}$ thỏa $a_{n+2}=\left\lfloor \frac{2a_n}{a_{n+1}} \right\rfloor+\left\lfloor \frac{2a_{n+1}}{a_n} \right\rfloor$.

Chứng minh tồn tại số nguyên dương $m$ sao cho $a_m=4$ và $a_{m+1} \in \{3;4 \}$.




#430638 $f(n+1)=2.3^{2^{n}}(f(n))^{2}-3^{2^...

Posted by dark templar on 25-06-2013 - 23:05 in Phương trình hàm

Tìm tất cả hàm số f xác định trên $\mathbb{N}^{*}$ thỏa điều kiện:

$\left\{\begin{matrix} f(1)=\frac{3\sqrt{3}}{2} \\ f(n+1)=2.3^{2^{n}}(f(n))^{2}-3^{2^{n}(n+1)} \end{matrix}\right.$

với mọi n thuộc $\mathbb{N}^{*}$

Xem ở đây.




#430633 Dãy số-Giới hạn Tuyển tập sưu tầm các bài toán từ Mathlinks.ro

Posted by dark templar on 25-06-2013 - 22:47 in Dãy số - Giới hạn

Dạng toán bài 34 ở trên VMF này có khá nhiều,trong box Dãy số-Giới hạn THPT và Olympiad. :) Tiếp tục :

 

Bài toán 36: Ta gọi $r(n)$ là tích của các thừa số nguyên tố phân biệt của số tự nhiên $n$.Cho dãy $\{a_n \}$ với $a_1$ cho trước và $a_{n+1}=a_n+r(a_n)$.Chứng minh rằng tồn tại hữu hạn số hạng liền kề của dãy này tạo thành 1 cấp số cộng.

-105-




#430628 $(x+2)log^2 _3 (x+1)+4(x+1)log_3 (x+1)-16=0$

Posted by dark templar on 25-06-2013 - 22:31 in Phương trình - hệ phương trình - bất phương trình

$(x+2)log^2 _3 (x+1)+4(x+1)log_3 (x+1)-16=0$

Điều kiện $x>-1$.

 

Đặt $t=\log_3 (x+1)$,ta có $(x+2)t^2+4(x+1)t-16=0$.

 

Xem đây là PT bậc 2 ẩn $t$,xét $\Delta'=4(x+1)^2+16(x+2)=4(x+3)^2$.Khi này $t_1=\frac{4}{x+2}$ và $t_2=-4$.

 

$(\star):t=\frac{4}{x+2} \iff (x+2)\log_3 (x+1)=4$

 

Đặt $u=\log_3 (x+1) \implies 3^{u}=x+1$,suy ra $u(3^{u}+1)=4 \iff 3^{u}(u-1)+3^{u}+u-4=0 \quad (*)$.

 

Dễ thấy $u=1$ là nghiệm của $(*)$.Xét $u>1$ thì $3^{u}+u-4>0$,suy ra $3^{u}(u-1)+3^{u}+u-4>0$,với $u<1$ cũng tương tự.

 

Vậy $u=1$ là nghiệm duy nhất của $(*) \iff x=2$.

 

$(\star):t=-4 \iff \log_3 (x+1)=-4 \iff x=3^{-4}-1$.

 

Kết luận: PT đã cho có 2 nghiệm là $\boxed{\displaystyle x \in \left\{\frac{-80}{81};2 \right\}}$.




#430617 $$\sum_{cyc}\frac{\sin^2 A}...

Posted by dark templar on 25-06-2013 - 21:58 in Bất đẳng thức - Cực trị

Bài toán 1: Với tam giác $ABC$ bất kỳ,ta có $\sum_{cyc}\frac{\sin^2 A}{\cos B\cos C}\ge\left(\frac{6r}{R}\right)^2$

 

Bài toán 2: Cũng với điều kiện như trên,chứng minh $\sum_{cyc}\frac{\cos^3 A}{\sin^2 B\sin^2 C} \ge \frac{2}{3}$.

 

 




#430567 Dãy số-Giới hạn Tuyển tập sưu tầm các bài toán từ Mathlinks.ro

Posted by dark templar on 25-06-2013 - 19:43 in Dãy số - Giới hạn



Bài toán 33: Cho dãy nguyên $\{a_n \}$ xác định bởi $ a_n = 2\cdot a_{n-1}+a_{n-2},\quad (n > 1),\quad a_0 = 0, a_1 = 1. $Chứng minh rằng $a_n$ chia hết cho $2^{k}$ khi và chỉ khi $n$ chia hết cho $2^{k}$.

-77-

Lời giải 1 bài toán 33

 

Các lời giải 2,3,4 là dành cho "$2^{k}|n$ thì $2^{k}|a_n$".

Lời giải 2 bài toán 33

Lời giải 3 bài toán 33

Lời giải 4 bài toán 33

 

Ta có 2 bài toán đề nghị sau:

 

Bài toán 33.1: Chứng minh rằng $n$ nguyên tố $\iff a_n$ nguyên tố.

 

Bài toán 33.2: Với mỗi $m>0$ và $0 \le j \le m$,ta có $2a_m|a_{m+j}+(-1)^{j}a_{m-j}$.

 

====================

Đề mới:

 

Bài toán 34: Cho phương trình $ x^n=x^{n-1}+...+x+1 $.

  1. Chứng minh rằng PT trên có duy nhất nghiệm thực dương,gọi nghiệm đó là $x_n$.
  2. Chứng minh $\lim_{n \to +\infty}x_n=2$
  3. Tính $\lim_{n \to +\infty}n(2-x_n)$.

Bài toán 35: Cho dãy $\{u_n \}$ thỏa $u_1=20;u_2=30$ và $u_{n+2}=3u_{n+1}-u_{n},n \ge 1$.Tìm $n$ để $1+5u_{n}u_{n+1}$ là số chính phương.

-99-




#430557 Dãy số-Giới hạn Tuyển tập sưu tầm các bài toán từ Mathlinks.ro

Posted by dark templar on 25-06-2013 - 19:04 in Dãy số - Giới hạn

Bài 33:

(2) => (1) : NX: $n\vdots 2^{k}$ thì $a_{n}\vdots a_{2^{k}}$ nên ta chỉ cânf cm $a_{2^{k}}\vdots 2^{k}$

Ta tìm đc cttq dãy sô: $a_{n}=\frac{(1+\sqrt{2})^{n}-(1-\sqrt{2})^n}{2\sqrt2}$

$\Rightarrow a_{2^{k}}=a_{2^{k-1}}((1+\sqrt2)^{2^{k-1}}+(1-\sqrt2)^{2^{k-1}})$

Chú ý: $(1+\sqrt2)^{2^{k-1}}+(1-\sqrt2)^{2^{k-1}}=a+b\sqrt2+a-b\sqrt2=2a\vdots 2$

nên = quy nạp ta dê dàng cm đc $a_{2^{k}}\vdots 2^{k}$

 

(1) => (2)  :

Ta quy nạp theo k

Giả sử đã đúng đến k, ta cm đúng với k+1

Đăt $n=2^{k+1}u+v$

Ta có $a_{n}=a_{2^{k+1}u+v}=((1+\sqrt2)^v+(1-\sqrt2)^v)a_{2^{k+1}u}+(-1)^va_{2^{k+1}u-v}$

$\Rightarrow a_{2^{k+1}u-v}\vdots 2^{k+1}$

theo gt quy nạp thì $2^{k+1}u-v\vdots 2^k$ nên nếu v khác 0 thì $v=2^k$

$\Rightarrow 2^{k+1}u-v=2^k(2u-1)$, vô lý.

do đó v=0 và ta có đpcm

Lời giải này chưa thực sự đầy đủ lắm :P Chỗ nhận xét $(2) \implies (1)$ ,với $n=m2^{k}$ thì ít nhất bạn cũng phải chứng minh $\sum_{j=0}^{m-1}(1+\sqrt{2})^{m-j}(1-\sqrt{2})^{j}$ là số nguyên thì mới kết luận được $a_{2^{k}}|a_n$.

 

Còn khúc công thức $a_{n}=a_{2^{k+1}u+v}=((1+\sqrt2)^v+(1-\sqrt2)^v)a_{2^{k+1}u}+(-1)^va_{2^{k+1}u-v} \quad (*)$,bạn cũng nên giải thích đầy đủ rõ ràng 1 xíu,thật ra nó phải là $2a_{2^{k+1}u}((1+\sqrt2)^v+(1-\sqrt2)^v)-(-1)^{v}a_{2^{k}u-v}$  ;),nhưng dù sao cũng hiểu được tư tưởng của lời giải.

 

Tuy nhiên mình cũng nhắc bạn là nên trình bày đầy đủ,rõ ràng lời giải để khi chúng mình tổng hợp sẽ dễ dàng hơn.

 

====================

Thật ra bài toán 33 này liên quan đến 1 dãy số rất nổi tiếng là dãy số Pell và các số Pell-Lucas.Dãy số $\{a_n \}$ ở trên chính là dãy số Pell.

 

Công thức $(*)$ mà cậu xài là xuất phát từ mối liên hệ giữa số Pell và số Pell-Lucas:

$$P_{m+n}=2P_{m}Q_{n}-(-1)^{n}P_{m-n}$$

 

Trong đó $P_{n}$ là số Pell thứ $n$ và $Q_{n}$ là số Pell-Lucas thứ $n$.

Tất nhiên còn nhiều đẳng thức liên quan khác nữa,có thể tìm thêm ở đây và ở đây.

 

Còn 1 tính chất nữa của bài 33 là $a_n$ nguyên tố khi và chỉ khi $n$ nguyên tố. :)




#430543 Đa thức -Tuyển tập các bài toán sưu tầm từ Mathlinks.ro

Posted by dark templar on 25-06-2013 - 18:07 in Đa thức



Bài toán 24: Chứng minh rằng với mỗi số nguyên dương $n$,tồn tại các đa thức $P(x)$ có bậc $n$ và $Q(x)$ có bậc $n-1$ thỏa $ (P(x))^2-1=(x^2-1)(Q(x))^2 $

-67-

Lời giải bài toán 24

 

@Đạt

 

Bài toán 25: Giả sử đa thức $P \in \mathbb{R}[x]$ thỏa mãn $P(0)=1$ và với $z \in \mathbb{C}$ có $|z|=1$ thì $|P(z)|=1$.Chứng minh rằng $P(z)=1;\forall z \in \mathbb{C}$.

-69-




#430499 Dãy số-Giới hạn Tuyển tập sưu tầm các bài toán từ Mathlinks.ro

Posted by dark templar on 25-06-2013 - 15:05 in Dãy số - Giới hạn

bài 33 mình nghĩ phải là chia hêt chư không phải chia hêt cho vì $a_{4}=12$ :-?

Cảm ơn cậu vì đã nhắc :P Đúng là "$a_n$ chia hết $2^{k}$ khi và chỉ khi $n$ chia hết $2^{k}$". Sẽ sửa lại đề...

 

Ẩn và hiện nội dung như thế nào vậy mọi người  :blink:

...

Bây giờ dùng quy nạp ta chứng minh dãy trên là số nguyên

 

Mình đã đưa cho cậu code biểu diễn rồi mà ? Còn việc chứng minh cho dãy $\{a_n \}$ là dãy nguyên trong bài 32 thì mới là bước quan trọng chứ,mong cậu đưa lời giải đầy đủ.Nếu được thì hãy đọc bài post đầu tiên của topic này để biết quy định trong topic.

 




#430485 Dạng tổng quát của đẳng thức $\sum_{k=0}^{\inft...

Posted by dark templar on 25-06-2013 - 13:50 in Dãy số - Giới hạn

Bài toán: Chứng minh rằng $\sum_{k=0}^{\infty}\frac{1}{F_{2^{k}}}=\frac{7-\sqrt{5}}{2} \quad (*)$

Trong đó $F_{n}$ là số Fibonacci thứ $n$.

 

Từ bài toán này,ta có thể đi đến bài toán sau:

 

Tổng quát: Cho số hữu tỷ dương $x$,chứng minh đẳng thức sau:

\[\sum\limits_{k = 0}^\infty  {\frac{1}{{\frac{1}{{\sqrt {{x^2} + 4} }}\left( {{{\left( {\frac{{x + \sqrt {{x^2} + 4} }}{2}} \right)}^{{2^k}}} - {{\left( {\frac{{x - \sqrt {{x^2} + 4} }}{2}} \right)}^{{2^k}}}} \right)}}}  = 1 + \frac{2}{x} + \frac{{x - \sqrt {{x^2} + 4} }}{2}\]

 

Đẳng thức $(*)$ có được khi chọn $x=1$.




#430478 $\frac{x+3}{(x+1)^{2}}+\frac...

Posted by dark templar on 25-06-2013 - 13:31 in Bất đẳng thức và cực trị

Cho số thực dương x,y,z thỏa mãn:$xyz=1$

CMR:

$\frac{x+3}{(x+1)^{2}}+\frac{y+3}{(y+1)^{2}}+\frac{z+3}{(z+1)^{2}}\geq 3$

Xem trong topic này,có khá nhiều cách giải :)




#430271 $6 \leq \sum \frac{x+y}{z}\leq...

Posted by dark templar on 24-06-2013 - 17:33 in Bất đẳng thức và cực trị



Một bài tương tự :P Dạng này

 

Bài toán: Cho số thực $a;b;c \in \left [ \frac{1}{2};2 \right ]$. Chứng minh rằng

 

$$\left ( a+b+c \right )\left ( \frac{1}{a}+\frac{1}{b}+\frac{1}{c} \right ) \leq \frac{225}{16}$$

Tổng quát:

Cho $n$ số thực $a_{i}(i=\overline{1,n})$ thỏa $a_{i} \in \left[a;\frac{1}{a} \right]$ với $0<a<1$ và $t_1+t_2+...+t_n=1(t_{i}>0;\forall i=1,2,...,n)$.Chứng minh:

$$\left(t_1a_1+t_2a_2+...+t_na_n \right)\left(\frac{t_1}{a_1}+\frac{t_2}{a_2}+...+\frac{t_n}{a_n} \right) \le \frac{1}{4}\left(a+\frac{1}{a} \right)^2$$




#430268 Đa thức -Tuyển tập các bài toán sưu tầm từ Mathlinks.ro

Posted by dark templar on 24-06-2013 - 17:14 in Đa thức

Bài toán 20: Cho $F$ là tập các đa thức $\Gamma$ có hệ số nguyên và PT $\Gamma(x)=1$ có nghiệm nguyên.Cho trước 1 số nguyên dương $k$,tìm giá trị nhỏ nhất của $m(k)>1$ thỏa mãn tồn tại $\Gamma \in F$ sao cho PT $\Gamma (x)=m(k)$ có đúng $k$ nghiệm nguyên phân biệt.

Lời giải bài toán 20

 

Đề mới:

 

Bài toán 23: Cho $p$ là số nguyên tố và $a \in \mathbb{Z}$.Chứng minh nếu đa thức $x^{p}-a$ khả quy thì $a=b^{p}$ với $b$ là 1 số nào đó.

 

Bài toán 24: Chứng minh rằng với mỗi số nguyên dương $n$,tồn tại các đa thức $P(x)$ có bậc $n$ và $Q(x)$ có bậc $n-1$ thỏa $ (P(x))^2-1=(x^2-1)(Q(x))^2 $

-67-

 

====================

Chú ý: Kể từ giờ,các bạn tham gia topic nếu post lời giải quá dài thì hãy đặt nó trong cặp thẻ sau:

[hide="Lời giải bài toán"]<Nội dung>[/hide]

Kết quả :

Lời giải bài toán
để load trang topic và công thức Toán nhanh hơn và tăng thẩm mỹ cho topic.




#430262 Dãy số-Giới hạn Tuyển tập sưu tầm các bài toán từ Mathlinks.ro

Posted by dark templar on 24-06-2013 - 16:51 in Dãy số - Giới hạn

Bài toán 30: Cho dãy $\{a_n \}$ thỏa $a_1>0,a_2>0$ và $a_{n+2}=\frac{2}{a_{n+1}+a_n}$.Chứng minh dãy có giới hạn hữu hạn và tìm giới hạn đó.
 
Bài toán 31: Cho dãy $\{a_n \}$ định bởi $a_1=a_2=1$ và $a_{n+1}=a_n+\frac{a_{n-1}}{n(n+1)}$.Chứng minh dãy có giới hạn hữu hạn.

Lời giải bài toán 30

Lời giải bài toán 31

 

Đề mới:

 

Bài toán 32: Chứng minh rằng chỉ tồn tại duy nhất 1 dãy nguyên $a_1;a_2;...;a_n$ thỏa $a_1=1;a_2>1$ và $ (a_{n+1})^{3}+1=a_{n}a_{n+2} $.

 

Bài toán 33: Cho dãy nguyên $\{a_n \}$ xác định bởi $ a_n = 2\cdot a_{n-1}+a_{n-2},\quad (n > 1),\quad a_0 = 0, a_1 = 1. $Chứng minh rằng $a_n$ chia hết cho $2^{k}$ khi và chỉ khi $n$ chia hết cho $2^{k}$.

-77-

 

====================

Chú ý: Kể từ giờ,các bạn tham gia topic nếu post lời giải quá dài hãy đưa nó vào giữa cặp lệnh như thế này:

[hide="Lời giải bài toán"]<Nội dung>[/hide]

Kết quả:

Lời giải bài toán
để cho load trang ,topic và công thức Toán cho nhanh hơn và tăng thẩm mỹ cho topic.




#430206 CM $S_n$ là số chính phương với $S_{n+1}=14S_{n...

Posted by dark templar on 24-06-2013 - 13:20 in Dãy số - Giới hạn

Ngoài ra tuy không hay cho lăm nhưng mình có thê làm thế này.

 

Dễ dàng tính được $S_2=3^2, S_3=11^2, S_4=41^2$ từ đây mình sẽ chứng minh $S_{n+2}=(4S_{n+1}-S_n)^2 \quad (*)$ ( bằng quy nạp)

Sao em chứng minh quy nạp cho đẳng thức $(*)$ được nhỉ ?  :ohmy: Nó là 1 đẳng thức sai,thử ngay với $n=1$ sẽ thấy.

 

Thực chất đẳng thức đó phải là $S_{n+2}=\left(4\sqrt{S_{n+1}}-\sqrt{S_{n}} \right)^2$ và nó cũng không khác với lời giải của anh là mấy. :luoi:




#430203 BĐT -Tuyển tập các bài toán sưu tầm từ Mathlinks.ro

Posted by dark templar on 24-06-2013 - 13:11 in Bất đẳng thức - Cực trị

Bài toán 29: Trong $2013$ số thực cho sẵn bất kỳ,chứng minh rằng sẽ có ít nhất 1 cặp số $(a;b)$ thỏa mãn BĐT $\frac{|1-ab||a-b|}{(a^2+1)(b^2+1)}<\frac{1}{2012}$.

 

Bài toán 30: Cho $2n$ số thực $a_1,a_2,...,a_n,b_1,b_2,...,b_n$ thỏa mãn $1 \ge b_1 \ge b_2 \ge ...\ge b_n \ge 0$.Chứng minh rằng tồn tại 1 số tự nhiên $k \le n$ sao cho $ |a_1b_1+a_2b_2+...+a_nb_n|\le |a_1+a_2+...+a_k| $

Lời giải bài toán 29

Lời giải bài toán 30

 

Đề mới:

 

Bài toán 31: Cho hàm $f$ là 1 đơn ánh từ $\mathbb{N^*}$ đến $\mathbb{N^*}$.Chứng minh với mọi $n$ thì $ \sum_{k=1}^{n}f(k)k^{-2}\geq\sum_{k=1}^{n}k^{-1}. $

 

Bài toán 32: Tìm số nguyên $k$ nhỏ nhất thỏa $ \sqrt{1+\sqrt{2+\sqrt{3+\ldots+\sqrt{2007}}}}<k $.

 

1 BĐT mạnh hơn là $ \sqrt{k+\sqrt{k+1+\sqrt{k+2+...+\sqrt{k+n}}}}<k+1 $

-218-

 

Nhận xét

 

====================

Chú ý: Kể từ giờ,các bạn tham gia topic nếu post lời giải quá dài thì hãy đặt nó trong cặp thẻ sau:

[hide="Lời giải bài toán"]<Nội dung>[/hide] 

Kết quả :

Lời giải bài toán
để load trang topic và công thức Toán nhanh hơn và tăng thẩm mỹ cho topic.

 




#430189 Một số bài toán tính tổng chọn lọc

Posted by dark templar on 24-06-2013 - 11:36 in Các dạng toán khác



Bài toán 41:

Cho dãy Fibonacci $\{F_n\}_{n\ge 1}$

Tính tổng:

$S_n=\sum_{k=1}^n \left\lfloor\frac{F_{2k}}{F_{k+1}}\right\rfloor$

Hướng giải bài toán 41

Lời giải bài toán 41

 

Em nghĩ là khi đưa ra lời giải của 1 bài toán trong topic này,ta hãy để nó dưới dạng "Ẩn-Hiện" như trên để khi load trang và công thức Toán sẽ nhanh hơn,giống như các topic Marathon bên ML vậy. :P

 

Bài toán 40




#430024 Russia MO 2001

Posted by dark templar on 23-06-2013 - 18:38 in Tổ hợp và rời rạc

Trong mỗi ô của một cái bảng có n hàng và 10 cột, có một chữ số được viết. Biết rằng cứ mỗi hàng A nào đó và 2 cột bất kì, ta luôn tìm được (chỉ) một hàng A có những con số khác với A tại điểm giao nhau với 2 cột. Chứng minh rằng : $n\geq 512$

p/s:  ko biết mình đặt "chỉ" (only) ở đó có đúng ko?  :unsure:

Nên dịch là "ta luôn tìm được 1 hàng có những chữ số khác với hàng $A$ khi mà hàng đó giao với 2 cột kể trên." :P